Pulmonary - Part 2

Approved & Edited by ProProfs Editorial Team
The editorial team at ProProfs Quizzes consists of a select group of subject experts, trivia writers, and quiz masters who have authored over 10,000 quizzes taken by more than 100 million users. This team includes our in-house seasoned quiz moderators and subject matter experts. Our editorial experts, spread across the world, are rigorously trained using our comprehensive guidelines to ensure that you receive the highest quality quizzes.
Learn about Our Editorial Process
| By Chachelly
C
Chachelly
Community Contributor
Quizzes Created: 513 | Total Attempts: 597,125
Questions: 25 | Attempts: 991

SettingsSettingsSettings
Pulmonary Quizzes & Trivia

Questions and Answers
  • 1. 

    A 55-year-old woman has had fever and dyspnea for a month along with a 2 kg weight loss. On physical examination her temperature is 37.7 C. A chest radiograph shows a reticulonodular pattern along with prominent hilar lymphadenopathy. A transbronchial lung biopsy is performed, and microscopic examination shows no viral inclusions, no fungi, no acid fast bacilli, and no atypical cells. Which of the following diseases is she most likely to have?

    • A.

      Silicosis

    • B.

      Sarcoidosis

    • C.

      Asbestosis

    • D.

      Tuberculosis

    • E.

      Usual interstitial pneumonitis

    Correct Answer
    B. Sarcoidosis
    Explanation
    (B) CORRECT. The symptoms and signs are classic. Sarcoidosis often involves the hilar lymph nodes.
    (A) Incorrect. Silicosis produces an interstitial lung disease with a reticulonodular pattern with silicotic nodules.
    (C) Incorrect. Pleural plaques are a typical finding for asbestosis.
    (D) Incorrect. Special stains on the lung biopsy would have shown acid fast bacilli.
    (E) Incorrect. UIP is a form of restrictive lung disease with interstitial fibrosis.

    Rate this question:

  • 2. 

    A 55-year-old man has been a cigarette smoker for the past 39 years. He has noted some blood-streaked sputum on coughing during the past week. He also has back pain. A chest radiograph shows a small 3 cm right hilar mass with several 1 to 2 cm peripheral lung nodules. A bone scan reveals multiple areas of increased uptake in the vertebrae, ribs, and pelvis. A sputum cytology reveals the presence of clusters of small cells having hyperchromatic nuclei and almost no cytoplasm. Which of the following laboratory test findings is he most likely to have as a consequence of his lung disease?

    • A.

      Positive antinuclear antibody

    • B.

      Platelet count of 55,000/microliter

    • C.

      Plasma cortisol at 8 am of 5 microgm/dL

    • D.

      Serum sodium of 113 mmol/L

    • E.

      Hyperuricemia

    Correct Answer
    D. Serum sodium of 113 mmol/L
    Explanation
    (D) CORRECT. He has a small cell anaplastic carcinoma with widespread metastases. The syndrome of inappropriate ADH producing hyponatremia is one form of paraneoplastic syndrome seen with this particular carcinoma.
    (B) Incorrect. Thrombocytopenia is not a feature of a paraneoplastic syndrome.
    (C) Incorrect. One form of paraneoplastic syndrome is Cushing syndrome from ectopic ACTH production, which should elevate the serum cortisol.
    (E) Incorrect. The turnover rate of lung cancers is generally not high enough to elevate the uric acid.

    Rate this question:

  • 3. 

    A 40-year-old woman has had a high fever for a week, accompanied by a cough productive of yellowish sputum. On physical examination her temperature is 38.2 C. There are diffuse rales in all lung fields. Her chest radiograph reveals patchy infiltrates in all lung fields, and there is a 4 cm rounded area of consolidation in the left upper lobe that has an air-fluid level. Examination of her sputum reveals numerous neutrophils. Which of the following infectious agents is most likely causing her pulmonary disease?

    • A.

      Staphylococcus aureus

    • B.

      Aspergillus niger

    • C.

      Mycobacterium tuberculosis

    • D.

      Mycoplasma pneumoniae

    • E.

      Adenovirus

    Correct Answer
    A. Staphylococcus aureus
    Explanation
    A) CORRECT. Pulmonary abscesses typically have an air-fluid level. More virulent bacterial organisms such as Staphylococcus aureus are likely to cause such a complication of a bronchopneumonia.
    (B) Incorrect. Fungus balls are usually firm masses that do not have central necrosis and/or cavitation.
    (C) Incorrect. Although cavitation is a feature of secondary tuberculosis, air-fluid levels within the cavities are not frequent (due to the caseous nature of the necrosis).
    (D) Incorrect. Infection with Mycoplasma pneumoniae typically produces patchy infiltrates, but abscess formation is not likely to occur, and the process is more interstitial with mononuclear cells.
    (E) Incorrect. Adenovirus produces an interstitial pneumonitis. However, viral pneumonias may be complicated by secondary bacterial pneumonias.

    Rate this question:

  • 4. 

    At autopsy, a 60-year-old man is found to have a peripheral 7 cm area of golden-yellow consolidation on sectioning of the left lung. Microscopically, this area has alveoli filled with foamy macrophages. Which of the following conditions involving his lung is most likely to be responsible for this finding?

    • A.

      Mycoplasma pneumoniae infection

    • B.

      Cystic fibrosis

    • C.

      Adenocarcinoma

    • D.

      Silicosis

    • E.

      Squamous cell carcinoma

    • F.

      Malignant mesothelioma

    • G.

      Thromboembolism

    Correct Answer
    E. Squamous cell carcinoma
    Explanation
    (E) CORRECT. Most squamous cell carcinomas are located centrally and arise in bronchi, leading to obstruction of a large airway that can cause a lipid pneumonia. In this case, the breakdown of lung tissue distal to the mass yields an 'endogenous lipid pneumonia.'
    (A) Incorrect. Mycoplasma infection produces an atypical pneumonia with interstitial infiltrates. Obstruction of airways is unlikely.
    (F) Incorrect. Mesotheliomas arise in the pleura and are peripheral masses unlikely to obstruct proximal bronchi to produce a lipid pneumonia.
    (C) Incorrect. Most adenocarcinomas are peripheral and do not arise within a large bronchus.
    (G) Incorrect. Thromboemboli obstruct the pulmonary arterial system, not the bronchi.
    (D) Incorrect. The silicotic nodules are small (a few millimeters) and do not obstruct bronchi.

    Rate this question:

  • 5. 

    A 12-year-old girl has the acute onset of dyspnea and wheezing. She coughs up a large, thick mucus plug. She has experienced similar previous episodes for the past 7 years, all of which lasted 1 to 6 hours. On physical examination her vital signs include T 37.1 C, P 109/minute, RR 27/minute, and BP 90/60 mm Hg. There are decreased breath sounds in all lung fields. A chest radiograph reveals hyperinflation but no infiltrates. Laboratory studies show WBC count of 8300/microliter with differential count of 60 segs, 3 bands, 16 lymphs, 10 monos, and 11 eosinophils. A sputum sample examined microscopically has increased numbers of eosinophils. Which of the following is the most likely diagnosis?

    • A.

      Mycoplasma pneumonia infection

    • B.

      Cystic fibrosis

    • C.

      Hypersensitivity pneumonitis

    • D.

      Bronchial asthma

    • E.

      Aspiration of gastric contents

    Correct Answer
    D. Bronchial asthma
    Explanation
    (D) CORRECT. In an acute asthmatic episode, there can be an outpouring of mucus which, along with some dehydration, can lead to the formation of mucus plugs. The asthmatic episodes are usually initiated by a type I hypersensitivity reaction, typically with exposure to an allergen such as pollen from goldenrod or other flowering plant. Up to 10% of children may be affected to some degree.
    (B) Incorrect. With cystic fibrosis, there is bronchiectasis and multiple infections, so the sputum is often purulent.
    (C) Incorrect. Extrinsic allergic alveolitis is produced, which is mainly an interstitial process. Extrinsic allergic alveolitis, or hypersensitivity pneumonitis, is acutely mediated by activation of complement from exposure to an allergen, but there is usually no eosinophila and the symptoms last longer than for asthma.

    Rate this question:

  • 6. 

    A 40-year-old previously healthy woman, a non-smoker, has had episodes of fever, non-productive cough, and dyspnea over the past 3 months. Her symptoms disappeared after a month's vacation, but reappeared when she returned home to take care of her canaries. On physical examination there are no abnormal findings. A chest radiograph shows fine diffuse and nodular infiltrates in all lung fields. Her disease is most likely to be produced via which of the following mechanisms?

    • A.

      Mast cell degranulation

    • B.

      Progressive interstitial fibrosis

    • C.

      Antigen-antibody complex formation

    • D.

      Langerhans cell proliferation

    • E.

      Infection with Mycobacterium kansasii

    Correct Answer
    C. Antigen-antibody complex formation
    Explanation
    (C) CORRECT. Birds make a lot of organic dust from their feathers. The result is an extrinsic allergic alveolitis. Getting away from the antigen (such as a vacation) will improve the situation. This is a form of type 3 hypersensitivity.
    (A) Incorrect. Asthma produces bronchoconstriction for short periods of time. Infiltrates should not be present. Extrinsic asthma is a form of type 1 hypersensitivity reaction, with allergen attaching to IgE bound to mast cells.
    (B) Incorrect. Interstitial fibrosis is a progressive process leading to restrictive lung disease. The course is not episodic.
    (D) Incorrect. In adults, the most common form of Langerhans cell histiocytosis is eosinophilic granuloma of lung, a process that produces larger nodules and is not episodic.
    (E) Incorrect. Tuberculosis in an adult is typically the reactivation or reinfection type with upper lobe granulomas. Pulmonary disease caused by M. kansasii resembles that caused by M. tuberculosis in most cases. The cell-mediated immune response to tuberculosis is a form of type 4 hypersensitivity.

    Rate this question:

  • 7. 

    A 63-year-old woman has had increasing dyspnea for 5 years. On physical examination her lungs are hyper-resonant without dullness. Tactile vocal fremitus is slightly decreased over all lung fields. Scattered expiratory wheezes and inspiratory rhonchi are present without basal crackles. She has a chest radiograph that reveals increased lung volumes and flattening of the diaphragmatic leaves. Spirometry demonstrates an FEV1 that is decreased more than the FVC so that the FEV1/FVC ratio is less than 70% of normal. Which of the following inhaled substances, which increases the elaboration of neutrophil elastase, is most likely to cause her pulmonary disease?

    • A.

      Chlorine

    • B.

      Silica

    • C.

      Carbon monoxide

    • D.

      Nicotine

    • E.

      Carbon

    Correct Answer
    D. Nicotine
    Explanation
    (D) CORRECT. She has findings of emphysema, and smoking is the most likely underlying cause. The nicotine in the cigarette smoke is chemotactic for neutrophils, and cigarette smoke activates the alternative complement pathway, releasing more mediators for neutrophil recruitment. Neutrophil elastase can damage the lung parenchyma. Though neutrophils are not numerous in the lung with emphysema, the cumulative effect of even small numbers of neutrophils over many years leads to the tissue damage.
    (A) Incorrect. Exposure to toxic gases produces an acute pneumonitis with injury to epithelium and edema.
    (B) Incorrect. Silica dust produces nodules that diminish lung volumes and lead to a restrictive lung disease.
    (C) Incorrect. CO poisoning has no significant effect on the lung, but CO binds more avidly to hemoglobin and leads to hypoxia.
    (E) Incorrect. Carbon dust leads to anthracosis, which is a benign process that virtually everyone on earth now develops. Large quantities of coal dust can produce progressive massive fibrosis, a restrictive lung disease.

    Rate this question:

  • 8. 

    A 54-year-old man has had increasing dyspnea for the past 6 years, but no cough. On physical examination there is increased jugular venous distension. He is afebrile. A chest radiograph shows increased lucency in upper lung fields and increased lung volumes, with flattening of the diaphragmatic leaves. There are no infiltrates. The pulmonary arteries are enlarged and prominent bilaterally, and his right heart border is enlarged. Which of the following pathologic findings is most likely to be present in his main pulmonary arteries?

    • A.

      Granulomatous vasculitis

    • B.

      Organizing thromboemboli

    • C.

      Medial dissection

    • D.

      Atherosclerosis

    • E.

      Aneurysm formation

    Correct Answer
    D. Atherosclerosis
    Explanation
    (D) CORRECT. Cor pulmonale is the result of pulmonary hypertension, which is the major cause for pulmonary atherosclerosis. His emphysema has reduced the pulmonary vascular bed, promoting the hypertension. Note that the forces driving systemic atherosclerosis are not operative on the pulmonary arterial system.
    (A) Incorrect. Wegener granulomatosus is a process that can lead to pulmonary vasculitis, though the peripheral arteries are typically involved, and more of a restrictive lung disease develops.
    (B) Incorrect. Pulmonary thromboembolism is an acute process. It is not related to emphysema.
    (C) Incorrect. A dissection does not occur in pulmonary arteries, as in the aorta, because the pressures are not as high, even with pulmonary hypertension.
    (E) Incorrect. Aneurysms may complicate atherosclerosis, but the degree of atherosclerosis and the arterial pressures in the pulmonary system are not as marked as in the aorta.

    Rate this question:

  • 9. 

    A 44-y/o F, non-smoker, has had a fever and cough for the past 4 days. She does not have hemoptysis or  weight loss, malaise, nausea, or vomiting. On physical examination her temperature is 37.6 C. There are decreased breath sounds over the right upper lung. A radiograph reveals a 6 cm area of infiltrates in the right upper lobe. She is given a course of antibiotic therapy, but her ough persists. A month later her chest x-ray now reveals a 3 cm peripheral mass in the right upper lobe. Which of the following neoplasms is most likely to be present in this woman?

    • A.

      Squamous cell carcinoma

    • B.

      Small cell anaplastic carcinoma

    • C.

      Adenocarcinoma

    • D.

      Mesothelioma

    • E.

      Carcinoid tumor

    Correct Answer
    C. Adenocarcinoma
    Explanation
    (C) CORRECT. Peripheral lung cancers (adenocarcinoma and large cell carcinoma) show less of an association with smoking than central cancers (small cell and squamous cell carcinoma).
    (A) Incorrect. Squamous cell carcinomas are often associated with smoking. They are often bulky central masses.
    (B) Incorrect. Oat cell carcinomas have a close association with smoking. Such cancers are not common in nonsmokers. They often metastasize early, so that the primary site (usually in a central location) is small when discovered.
    (D) Incorrect. Asbestos exposure is a likely antecedent to pleural mesothelioma, which produces a bulky pleural mass.
    (E) Incorrect. Carcinoid tumors are neuroendocrine neoplasms on the opposite end of the spectrum from oat cell carcinomas. Carcinoids tend to form masses within the larger bronchi.

    Rate this question:

  • 10. 

    A 70-year-old woman has been bedridden for 5 weeks following a cerebrovascular accident (CVA). She has the sudden onset of dyspnea, but has no further symptoms until two days later when she experiences left sided pleuritic chest pain. A day later she suffers another CVA and dies. At autopsy, she is found to have a wedge-shaped area of hemorrhage based on the pleura of the left lower lobe. Which of the following pathologic findings in her pulmonary arterial branches is she most likely to have?

    • A.

      Atherosclerosis

    • B.

      Aspergillosis

    • C.

      Fat embolism

    • D.

      Vasculitis

    • E.

      Thromboembolism

    Correct Answer
    E. Thromboembolism
    Explanation
    (E) CORRECT. An embolus to a medium-sized arterial branch is not large enough to kill the patient, but large enough to cause an infarction. Her bedridden state predisposes her to deep venous thrombosis and thromboembolism--the CVAs are due to separate systemic arterial problems--or the systemic and pulmonary embolization can be tied together by a hypercoagulable state, or more remotely by a 'paradoxical' embolus through a patent foramen ovale once the right sided-pressures increased following the initial pulmonary thromboembolic event.
    (A) Incorrect. Although they are part of the systemic circulation, they supply little blood to lungs. They are small and not involved with systemic atherosclerosis. Their lack of occlusion with thromboembolization leads to hemorrhage into pulmonary infarcts.
    (D) Incorrect. A vasculitic process such as Wegener granulomatosis would be diffuse, not localized. The size of the arteries involved with most vasculitides is too small to cause an infarction, but could lead to pulmonary hypertension, however.

    Rate this question:

  • 11. 

    A 38-year-old previously healthy woman has had a worsening non-productive cough for the past 4 days. On physical examination her temperature is 38.3 C. A chest radiograph shows patchy infiltrates and diffuse interstitial markings. Laboratory studies show a sputum gram stain with mixed flora. Her Hgb is 12.9 g/dL, platelet count 229,450/microliter, and WBC count 5815/microliter. Her cold agglutinin titer is elevated. Following a course of erythromycin therapy, she improves, with no complications. Which of the following infectious agents is most likely to cause the pulmonary disease seen in this woman?

    • A.

      Nocardia asteroides

    • B.

      Mycoplasma pneumoniae

    • C.

      Mycobacterium kansasii

    • D.

      Respiratory syncytial virus

    • E.

      Chlamydia psittici

    • F.

      Adenovirus

    • G.

      Klebsiella pneumoniae

    Correct Answer
    B. Mycoplasma pneumoniae
    Explanation
    B) CORRECT. Mycoplasma affects the interstitium more, and is not an alveolar filling process. It is a cause for a 'primary atypical pneumonia' which is difficult to diagnose because this organism is not cultured by routine methods for bacterial organisms. The cold agglutinin titer is elevated in about half of cases and is a characteristic finding.
    (A) Incorrect. Nocardia braziliensisInfections occur in immunocompromised hosts and produce a chronic abscessing process.
    (C) Incorrect. M. kansasii produces a granulomatous disease similar to tuberculosis.
    (D) Incorrect. RSV infections are more likely to occur in children. They do not respond to erythromycin therapy.
    (E) Incorrect. Psitticosis is rare and can lead to a severe pneumonia known as ornithosis. It does not respond to erythromycin therapy.
    (F) Incorrect. Adenovirus can produce interstitial infiltrates, but does not respond to erythromycin.
    (G) Incorrect. Klebsiella is a gram negative bacterium that most often produces a bronchopneumonia with productive cough and neutrophilia.

    Rate this question:

  • 12. 

    A 23-year-old primigravida is found on prenatal testing to have an elevated hemoglobin A1C level. Her pregnancy is uncomplicated until the 29th week of gestation, when she has the onset of premature labor and delivers a male infant 24 hours later. The infant initially has Apgar scores of 4 and 6 at 1 and 5 minutes, but within an hour is in severe respiratory distress and requires intubation with mechanical ventilation. Which of the following pharmacologic therapies administered to the mother prior to birth could have helped to prevent this infant's neonatal respiratory distress?

    • A.

      Hydrocortisone

    • B.

      Nafcillin

    • C.

      Ibuprofen

    • D.

      Lecithin

    • E.

      Vitamin A

    • F.

      Surfactant

    Correct Answer
    A. Hydrocortisone
    Explanation
    (A) CORRECT. She has diabetes mellitus, which inhibits fetal lung development. At 29 weeks, the baby's lungs do not make sufficient surfactant. Corticosteroids administered to the mother help to speed up type II pneumonocyte production of surfactant in the baby. At birth, exogenous surfactant can be given to the neonate.
    (C) Incorrect. Non-steroidal anti-inflammatory drugs will not help to improve fetal lung maturity.
    (D) Incorrect. The lecithin-sphingomyelin (L/S) ratio is used as a measure of fetal lung maturity.
    (F) Incorrect. Exogenous surfactant administered to the infant soon after birth can help prevent respiratory distress syndrome with hyaline membrane disease.

    Rate this question:

  • 13. 

    For the past 5 months, a 51-year-old woman has noted increased swelling of her lower legs as the day progresses. She has no fever and no cough. On physical examination, she has pitting edema to the knees. A chest radiograph reveals bilateral pleural effusions, and the right heart border is prominent. Laboratory studies show a serum AST of 238 U/L, ALT 263 U/L, LDH 710 U/L, and CK 127 U/L. Which of the following underlying diseases is most likely to cause these findings?

    • A.

      Goodpasture syndrome

    • B.

      Recurrent thromboembolism

    • C.

      Renovascular hypertension

    • D.

      Bronchial asthma

    • E.

      Rheumatoid arthritis

    Correct Answer
    B. Recurrent thromboembolism
    Explanation
    (B) CORRECT. Pulmonary hypertension and right heart failure can occur in the small number of cases in which recurrent thromboembolism takes place. The right heart failure that occurs then leads to hepatic passive congestion with centrilobular necrosis that is the cause for the increased transaminases.
    (A) Incorrect. In Goodpasture syndrome there is antibody directed at vascular basement membrane, leading to acute hemorrhage.
    (C) Incorrect. Hypertension in the systemic circulation is not accompanied by pulmonary arterial hypertension with predominantly right-sided heart failure with cor pulmonale.
    (D) Incorrect. The episodes of bronchial asthma are short and typically do not lead to significant alteration to the pulmonary vasculature.
    (E) Incorrect. Rheumatoid arthritis may be accompanied by formation of rheumatoid nodules in various tissues, including lung, but they are usually peripheral and not numerous.

    Rate this question:

  • 14. 

    A 51-year-old man received an orthotopic cardiac transplant a month ago. He has developed a fever with cough over the past 5 days. On physical examination his temperature is 37.5 C. A chest CT scan shows consolidation with abscess formation involving the left lower lobe. A sputum gram stain reveals normal upper respiratory tract flora. He does not respond to antibiotic therapy over the next 6 months. His mental status deteriorates and MR imaging of the brain shows multiple abscesses. He is most likely to have an infection with which of the following organisms?

    • A.

      Mycoplasma pneumoniae

    • B.

      Aspergillus fumigatus

    • C.

      Mycobacterium avium-intracellulare

    • D.

      Nocardia braziliensis

    • E.

      Cytomegalovirus

    • F.

      Pneumocystis carinii (jiroveci)

    Correct Answer
    D. Nocardia braziliensis
    Explanation
    (D) CORRECT. Nocardia braziliensis infection can persist and lead to chronic abscesses. It can complicate the course of immunocompromised patients.
    (B) Incorrect. Aspergillus does not typically produce abscesses.
    (C) Incorrect. MAI typically affects AIDS patients, and lung involvement is minimal.
    (E) Incorrect. CMV does not produce abscesses.
    (F) Incorrect. Pneumocystis pneumonia is rarely disseminated and most often produces a diffuse pneumonic consolidation with minimal inflammatory cell infiltration and no abscess formation.

    Rate this question:

  • 15. 

    Three weeks after visiting her grandmother dying from a respiratory tract infection, a healthy 5-year-old girl develops a fever along with wheezing. On physical examination her temperature is 37.9 C. Her lung fields are clear to auscultation but there are expiratory wheezes. A chest radiograph reveals a solitary 2 cm peripheral mid-lung nodule and marked hilar lymphadenopathy. Laboratory studies show Hgb 13.6 g/dL, platelet count 183,600/microliter, and WBC count 5480/microliter. These findings are most consistent with infection by which of the following organisms?

    • A.

      Mycobacterium tuberculosis

    • B.

      Candida albicans

    • C.

      Coccidioides immitis

    • D.

      Aspergillus flavus

    • E.

      Bacteroides fragilis

    • F.

      Streptococcus pneumoniae

    • G.

      Respiratory syncytial virus

    Correct Answer
    A. Mycobacterium tuberculosis
    Explanation
    (A) CORRECT. The pattern of lung involvement is the classical 'Ghon complex' of primary tuberculosis, which is seen more commonly in children, though only about 5% of cases are symptomatic. The enlarged hilar nodes can impinge upon central airways to produce obstruction.
    (B) Incorrect. Candidiasis is seen in persons who are immunocompromised. A true Candida pneumonia is rare. Candida is more likely to colonize the upper respiratory tract.
    (C) Incorrect. Fungal granulomatous disease can mimic tuberculosis but is less common overall and is more typically seen in adults.
    (D) Incorrect. This is most likely to be seen in immunocompromised adults.
    (E) Incorrect. Lung abscesses from B. fragilis are not common in healthy children. Abscesses usually complicate a previous pulmonary infection or occur with aspiration.
    (F) Incorrect. Pneumococcal infections can produce a lobar pneumonia or bronchopneumonia.
    (G) Incorrect. RSV can produce childhood pneumonias, but they are typically interstitial pneumonias.

    Rate this question:

  • 16. 

    A 44-year-old previously healthy man has the sudden onset of severe dyspnea. On physical examination he is afebrile. There are absent breath sounds over the right lung fields. A chest x-ray shows pulmonary atelectasis involving all of the right lung. Which of the following conditions is most likely to produce these findings?

    • A.

      Aspiration of a foreign body

    • B.

      Pulmonary embolism

    • C.

      Squamous cell carcinoma

    • D.

      Penetrating chest trauma

    • E.

      Bronchiectasis

    Correct Answer
    D. Penetrating chest trauma
    Explanation
    (D) CORRECT. This would lead to pneumothorax with lung collapse. Such trauma is not infrequent, particularly in auto accidents.
    (A) Incorrect. Although atelectasis can occur distal to bronchial obstruction, it would be unusual for a foreign body to occlude a mainstem bronchus. The resorption of air is not immediate.
    (B) Incorrect. Occlusion of pulmonary arterial circulation does not collapse a lung.

    Rate this question:

  • 17. 

    A newborn male infant develops increasing respiratory distress within an hour following an uncomplicated vaginal delivery at 36 weeks gestation. A plain film radiograph reveals near opacification of both lungs. Despite intubation and positive pressure ventilation, the baby dies within two days. At autopsy, the infant's lungs demonstrate extensive pink hyaline membranes. Which of the following maternal conditions is most likely to increase the risk for this infant's respiratory distress?

    • A.

      Gestational diabetes

    • B.

      Hyperemesis gravidarum

    • C.

      Iron deficiency

    • D.

      Preeclampsia

    • E.

      Systemic lupus erythematosus

    Correct Answer
    A. Gestational diabetes
    Explanation
    (A) CORRECT. The hyperinsulinism in the baby as a result of the high glucose impedes development of the type II pneumonocytes. By 36 weeks there should normally be sufficient surfactant to prevent hyaline membrane disease. Tests for fetal lung maturity include the L/S ratio, fluorescence polarization (fpol), and phosphatidyl glycerol (PG).
    (B) Incorrect. Hyperememsis is 'morning sickness' which makes the mother feel miserable, but does not affect fetal lung maturity.
    (C) Incorrect. Iron deficiency leads to maternal and infantile anemia, but does not affect fetal lung maturity.
    (D) Incorrect. Preeclampsia may lead to premature delivery, but at 36 weeks this infant's lungs should have sufficient surfactant production.
    (E) Incorrect. SLE does not directly affect fetal lung maturity.

    Rate this question:

  • 18. 

    Following an acute pharyngitis lasting 4 days, a 10-year-old boy develops neck pain and marked halitosis. On physical examination is breath is very malodorous. A CT scan shows an abscess in the peritonsillar region. Laboratory studies include a culture of the abscess which grows anaerobic flora. Which of the following aerobic organisms is most likely to be cultured from his abscess?

    • A.

      Staphylococcus aureus

    • B.

      Hemophilus influenzae

    • C.

      Corynebacterium diphtheriae

    • D.

      Bordetella pertussis

    • E.

      Group A Streptococcus

    Correct Answer
    E. Group A Streptococcus
    Explanation
    (E) CORRECT. A peritonsillar abscess is usually a complication of a strep throat.
    (A) Incorrect. Staphylococcal infections of the pharynx are not common in children.
    (B) Incorrect. Hemophilus more typically produces an epiglottitis that may be obstructive. It can also lead to pneumonia and to meningitis.
    (C) Incorrect. Diptheria is rare nowadays due to immunization. In the acute disease there is a pseudomembrane that is formed, not an abscess.
    (D) Incorrect. This is the causative agent for whooping cough, which is rare because of immunization. There is a laryngotracheobronchitis, but rarely mucosal erosion.

    Rate this question:

  • 19. 

    A 65-year-old man has had no major medical problems prior to the past year, when he noted increasing malaise along with an 8 kg weight loss. He is a non-smoker. He currently does not have fever, cough, dyspnea, or any respiratory difficulties. On physical examination, he has non-tender supraclavicular lymphadenopathy. The lungs are clear to auscultation. A chest x-ray shows multiple solid nodules ranging from 1 to 3 cm scattered throughout all lung fields. No infiltrates or areas of consolidation are noted. Laboratory studies show Hgb 11.6 g/dL, Hct 34.7%, MCV 83 fL, and WBC count 6280/microliter. Which of the following pathologic processes in his lungs is most likely to account for these findings?

    • A.

      Pulmonary infarctions

    • B.

      Foreign body aspiration

    • C.

      Metastatic carcinoma

    • D.

      Nocardia asteroides infection

    • E.

      Silicosis

    Correct Answer
    C. Metastatic carcinoma
    Explanation
    (C) CORRECT. Multiple masses should suggest metastases, rather than a primary lung tumor. His lack of a cough or fever is against an infectious cause or aspiration.
    (D) Incorrect. Nocardiosis is typically seen in immunocompromised patients and leads to abscess formation.
    (E) Incorrect. Silicosis severe enough to produce lung nodules should lead to a restrictive lung disease with dyspnea.

    Rate this question:

  • 20. 

    A 43-year-old woman who does not smoke becomes increasingly dyspneic over 8 years' time. She does not have a cough or increased sputum production. She is afebrile. On physical examination she has decreased breath sounds with hyperresonance in all lung fields. A chest radiograph reveals increased lucency of all lung fields. Laboratory studies show her serum alpha-1-antitrypsin level is 18 mg/dL. Which of the following microscopic portions of the lung is most likely to be affected by her condition?

    • A.

      Lymphatic channel

    • B.

      Alveolar duct

    • C.

      Bronchial artery

    • D.

      Interstitium

    • E.

      Terminal bronchiole

    Correct Answer
    B. Alveolar duct
    Explanation
    B) CORRECT. Alpha-1-antitrypsin (AAT) deficiency leads to a panacinar form of emphysema which involves the distal acinus beyond the respiratory bronchiole.
    (A) Incorrect. The lung lymphatics are not directly affected by emphysema.
    (C) Incorrect. The bronchial arteries supply systemic blood to the lung, primarily in the intersitium, and are not directly affected by emphysematous processes.
    (D) Incorrect. The intersititium may appear more prominent with emphysema when there is collapse following loss of alveoli..
    (E) Incorrect. Alpha-1-antitrypsin deficiency leads to a panacinar form of emphysema with the bulk of the damage distal to the respiratory bronchiole.

    Rate this question:

  • 21. 

    A 30-year-old woman is in the 28th week of an uncomplicated pregnancy when she experiences the sudden onset of severe abdominal pain, followed by vaginal bleeding, then the onset of labor. A girl infant is delivered on the way to the hospital. On arrival within an hour, the baby is in respiratory distress and requires intubation and mechanical ventilation. A day later, a chest radiograph shows opacification of both lungs. The baby's respiratory status does not improve. Which of the following histopathologic findings is most likely to be present in this baby's lungs?

    • A.

      Neutrophilic exudates in the alveoli

    • B.

      Irregular fibrosis with airspace dilation

    • C.

      Decreased lamellar bodies in type II pneumocytes

    • D.

      Diffuse alveolar hemorrhage

    • E.

      Interstitial lymphocytic infiltrates

    Correct Answer
    C. Decreased lamellar bodies in type II pneumocytes
    Explanation
    (C) CORRECT. The baby has been born prematurely, with incomplete lung development, and the lack of pulmonary surfactant production leads to hyaline membrane disease with respiratory distress in the newborn.
    (A) Incorrect. A congenital pneumonia is unlikely to have developed in this rapid a time.
    (B) Incorrect. These are findings of bronchopulmonary dysplasia, which may occur later in the course.
    (D) Incorrect. Hemorrhage can occur with a coagulopathy in a newborn, but is unusual in this emergent situation.
    (E) Incorrect. Lymphocytic infiltrates suggest a viral process, but congenital pneumonias are typically caused by bacterial organisms

    Rate this question:

  • 22. 

    A 41-year-old woman has a 1 year history of episodic dyspnea. On physical examination there are expiratory wheezes. Her chest radiograph shows a few small 0.5 cm perihilar nodules. Laboratory studies show an elevated serum IgE along with peripheral blood eosinophilia. A sputum sample shows eosinophils. Which of the following pathologic findings is most likely present in her bronchi?

    • A.

      Non-invasive aspergillosis

    • B.

      Blastomycosis

    • C.

      Invasive candidiasis

    • D.

      Wegener granulomatosis

    • E.

      Cytomegalovirus

    Correct Answer
    A. Non-invasive aspergillosis
    Explanation
    (A) CORRECT. She has an allergic bronchopulmonary aspergillosis. Her asthma is exacerbated by a type I hypersensitivity reaction to the fungus in the bronchi.
    (B) Incorrect. Blastomycosis is an uncommon form of fungal disease that is likely to produce a granulomatous response.
    (C) Incorrect. Candidiasis is more likely to be seen in immunocompromised patients.
    (D) Incorrect. Wegener granulomatosis is not associated with asthma.

    Rate this question:

  • 23. 

    A male infant has initial Apgar scores of 5 and 6 at 1 and 5 minutes following birth by normal vaginal delivery at 30 weeks gestation. However, increasing respiratory distress in the next hour requires intubation and positive pressure ventilation. Two months later, the infant is finally taken off the ventilator, but still does not oxygenate normally. Which of the following diseases has this infant most likely developed?

    • A.

      Diffuse alveolar damage

    • B.

      Bronchial asthma

    • C.

      Bronchiectasis

    • D.

      Tracheo-esophageal fistula

    • E.

      Bronchopulmonary dysplasia

    Correct Answer
    E. Bronchopulmonary dysplasia
    Explanation
    (E) CORRECT. The BPD is a complication of the treatment for neonatal respiratory distress. The positive pressure ventilation with the higher FIO2's, and the prolonged intubation, all contribute. Generally, these are premature infants who had a respiratory complication following birth, such as hyaline membrane disease.
    (A) Incorrect. Diffuse alveolar damage (DAD) is an acute condition more likely to be seen in adults with severe lung injury.
    (B) Incorrect. Asthma is not typically a neonatal disease, and asthma tends to be episodic, without development of chronic lung disease.
    (C) Incorrect. Bronchiectasis is unusual at this age, because the bronchial obstruction and inflammation usually develop over months to years.
    (D) Incorrect. A T-E fistula is a congenital anomaly that can predispose to aspiration and pneumonia, not to immediate respiratory distress following birth.

    Rate this question:

  • 24. 

    A 6-year-old child has the sudden onset of dyspnea with wheezing. On physical examination he is afebrile but has absent breath sounds on the right. His temperature is 37 C, pulse 82/minute, respiratory rate 28/minute, and blood pressure 100/60 mm Hg. An arterial blood gas measurement shows pO2 95 mm Hg, pCO2 25 mm Hg, and pH 7.55. Following administration of 100% FiO2 by nasal canula, a repeat measurement shows pO2 95 mm Hg, pCO2 25 mm Hg, and pH 7.55. Which of the following is the most likely diagnosis?

    • A.

      Foreign body aspiration

    • B.

      Bronchial asthma

    • C.

      Paraseptal emphysema

    • D.

      Thromboembolism

    • E.

      Carcinoid tumor

    Correct Answer
    A. Foreign body aspiration
    Explanation
    (A) CORRECT. An inhaled object could obstruct a bronchus completely, with air resorbtion and collapse of lung distal to the point of obstruction. This produces a shunt defect with a V/Q mismatch. Since there is perfusion but no ventilation, even 100% oxygen will not make a difference. His hyperventilation has acutely produced an uncompensated respiratory alkalosis.
    (B) Incorrect. Asthma can produce acute bronchoconstriction, but bilaterally.
    (C) Incorrect. Paraseptal emphysema can produce focal bullae that can rupture, with sudden pneumothorax, but this is not common in a child.
    (D) Incorrect. Thromboembolism affects the pulmonary arterial vasculature, not the airways.
    (E) Incorrect. A carcinoid tumor can obstruct a bronchus, but usually not acutely, and such a tumor is rare in a child.

    Rate this question:

  • 25. 

    A 60-year-old woman develops multiple organ failure 3 weeks following a pneumonia complicated by septicemia. Antibiotic therapy has resulted in sputum and blood cultures that are now without growth of organisms. Nevertheless, she requires intubation with mechanical ventilation, but it becomes progressively more difficult to maintain her oxygen saturations. Ventilatory pressures must be increased. A portable chest radiograph shows increasing opacification of all lung fields. Which of the following pathologic processes is most likely now to be present in her lungs?

    • A.

      Pulmonary arterial vasculitis

    • B.

      Diffuse alveolar damage

    • C.

      Extensive neutrophilic alveolar exudates

    • D.

      Extensive intra-alveolar hemorrhage

    • E.

      Widespread bronchiectasis

    Correct Answer
    B. Diffuse alveolar damage
    Explanation
    (B) CORRECT. Diffuse alveolar damage (DAD) is the pathologic term for adult respiratory distress syndrome (ARDS) that is the final common pathway for many acute lung injuries. DAD produces increasing interstitial thickening with mixed inflammation and features of an acute restrictive lung disease.
    (A) Incorrect. Multiple organ failure does not lead to vasculitis, which is not common in the lung.
    (C) Incorrect. With negative cultures, it is unlikely that her pneumonia is still present.
    (D) Incorrect. Diffuse alveolar hemorrhage can occur in persons with organ transplants and coagulopathies, but is not a major feature of multiple organ failure.
    (E) Incorrect. Bronchiectasis occurs over a longer period of time, with continuing inflammation and destruction of bronchi.

    Rate this question:

Quiz Review Timeline +

Our quizzes are rigorously reviewed, monitored and continuously updated by our expert board to maintain accuracy, relevance, and timeliness.

  • Current Version
  • Mar 20, 2023
    Quiz Edited by
    ProProfs Editorial Team
  • Jul 26, 2012
    Quiz Created by
    Chachelly
Back to Top Back to top
Advertisement
×

Wait!
Here's an interesting quiz for you.

We have other quizzes matching your interest.